LSAT and Law School Admissions Forum

Get expert LSAT preparation and law school admissions advice from PowerScore Test Preparation.

 Administrator
PowerScore Staff
  • PowerScore Staff
  • Posts: 8950
  • Joined: Feb 02, 2011
|
#40637
Complete Question Explanation
(The complete setup for this game can be found here: lsat/viewtopic.php?t=4425)

The correct answer choice is (D)

This is the one question in the game where Identifying the Templates would have provided some benefits, because seeing multiple solutions would have helped eliminate incorrect answer choices in this Cannot Be True question. However, that’s a minor benefit compared to the time required to draw out the templates.

Assuming you do not have the templates when you arrive at this question, and you do not instantly see an answer that you feel is correct, you have a choice of how to proceed:

1. ..... Review prior work in the hope that another question can provide insight into this question.
2. ..... Examine the rules again in the hope that there’s a logical path you overlooked.
3. ..... Pound through the question with brute force by trying hypotheticals for each answer choice until you arrive at ..... ..... ..... the one that cannot occur.
4. ..... Skip the question and guess at the correct answer.

Assuming that you do not want to skip the question (and thus that you have sufficient time to complete this question), and that you do not want to try to solve for each answer choice, your best bet is option 1 or option 2. Part of that choice depends on how you feel about the question stem—was there anything in the stem that you feel suggests a solution can be found through a rule review? If so, proceed by reconsidering the rules. If not, look at the work produced in other questions.

Let’s start with the rule review approach (option 2), because knowing the rules and their implications is usually the surest way to succeed on any question. The case referenced in the question stem is the Thompson case. From the third rule, H is assigned to Sicoli, so the first check would be to see if H appears in an answer choice (meaning H is assigned to Thompson), which would be a violation of the third rule and grounds for selecting that answer as correct. Sadly, that does not occur.

The next step is to consider where the Thompson case is mentioned directly in the rules. It appears in the first rule, which involves F assigned to Raimes and K assigned to Thompson in a double-arrow relationship. Thus, there seems to be a logical basis to analyze this question from the perspective of what alignment would cause a violation of this rule (since it is a Cannot Be True question). To create this violation, you would want K removed from the Thompson case (which narrows the focus to answer choices (A), (B), and (D)), but to still somehow force F to be assigned to Raimes (which, from the second rule, suggests you’d want G assigned to a case with another paralegal, which occurs in answer choice (D)). Since answer choice (D) seems promising, let’s take a closer look at it.

In (D), G and L are assigned to Thompson. Thus, G is not the sole paralegal assigned to a case, and thus from the second rule F must be the sole paralegal assigned to a case. However, that case cannot be Sicoli, since H is already assigned to Sicoli from the third rule. Thus, F has to be assigned to Raimes. This enacts the first rule, meaning that K must then be assigned to Thompson, However, in this answer choice, K is not assigned to Thomspon, and hence this answer choice cannot occur. Therefore, answer choice (D) is correct.

If the rule review isn’t your preferred approach, or if you attempt it and do not make worthwhile progress, then check prior work to see if you can eliminate any answers. Questions #18 and #21 provide solutions that have only F assigned to Thompson, but none of the answer choices can be eliminated by those hypotheticals.

Question #20 provides a partial solution, and from that solution a selection of K, L, and G/F could be the two paralegals assigned to Thompson, and if F is assigned to Raimes, then either K and L, or K and G could be the two paralegals assigned to Thompson, which, if you happened to examine question #20, would help eliminate answer choices (C) and (E) from consideration. However, most students don’t explore question #20 this deeply.

Thus, after a quick glance at prior work, most students have not eliminated any answer choices, and some have maybe eliminated (C) and (E). One method of attack would then be to jump ahead to question #23, in the hope that #23 would produce a solution that could be used to eliminate an answer choice (or choices) in question #22. And, ultimately, #23 can be used to eliminate answer choice (A). Note this strategy for the future—if you have a Global question that is not based on a full solution (like this question) and prior work does not solve the question fully (or eliminate enough answer choices for you to simply solve the problem at that point), then skip the question for the time being and solve the remaining questions, and then use the information from those questions to return to and solve the Global question.

Thus, at best, some students would have eliminated answer choices (A), (C), and (E), and they could attack the remaining two answers using hypotheticals.
 lsatstudier
  • Posts: 49
  • Joined: Oct 24, 2016
|
#32137
Hi,

I was wondering if someone could explain why the answer is D instead of B. Is B wrong because Laurie would never be by herself so we need to eliminate the answer? Wouldn't Gina and Laurie complete the assignment of paralegals on the case as seen in Question 21?

Thank you!
 Adam Tyson
PowerScore Staff
  • PowerScore Staff
  • Posts: 5387
  • Joined: Apr 14, 2011
|
#32147
Hi there lsatstudier, thanks for the question. Did you try completing the scenario in answer D to see what else would happen if G and L were the only ones assigned to T? That's the best way to test these answers, by following through and applying all the rules to see what else would have to happen.

In this case, if G and L are the only ones at T, then K is not at T. Following the first rule of the game, if K is not at T then F cannot be at R. That would mean F would have to go to S. The third rule has H at S, always, so F would not be alone there. Now we should see that we are being forced to violate the second rule of the game, requiring that either F or G must be alone in their group. G is with L, F is with H, and that won't work!

It is possible that G and L are the only two at their case, but when that happens it can only happen with them in group R. If that happens, then F has to be alone in group T, and H and K end up together in group S. In fact, if you do the mini diagram for question 21, you would likely have already tried G and L in group T and discovered it was impossible, making question 22 a breeze requiring no extra work.

L can be alone at T - again, just follow the rules from there. That would again mean that K cannot be there, so F cannot be at R, forcing F to S with H. Since one of F or G must always be alone, we then place G alone at R, and K ends up in S with H and F, a 1-3-1 scenario that is just fine with our rules.

Trying those answers out, testing things, is part of the diagramming process, and also a good way to tackle many local ("if") questions in games. Don't just think about it in your head - think with your pencil! That's the way to make easy inferences, eliminate losers and find winners.

Put that pencil to the paper and see what happens! Good luck!

Get the most out of your LSAT Prep Plus subscription.

Analyze and track your performance with our Testing and Analytics Package.